www.vorkurse.de
Ein Projekt von vorhilfe.de
Die Online-Kurse der Vorhilfe

E-Learning leicht gemacht.
Hallo Gast!einloggen | registrieren ]
Startseite · Mitglieder · Teams · Forum · Wissen · Kurse · Impressum
Forenbaum
^ Forenbaum
Status Mathe-Vorkurse
  Status Organisatorisches
  Status Schule
    Status Wiederholung Algebra
    Status Einführung Analysis
    Status Einführung Analytisc
    Status VK 21: Mathematik 6.
    Status VK 37: Kurvendiskussionen
    Status VK Abivorbereitungen
  Status Universität
    Status Lerngruppe LinAlg
    Status VK 13 Analysis I FH
    Status Algebra 2006
    Status VK 22: Algebra 2007
    Status GruMiHH 06
    Status VK 58: Algebra 1
    Status VK 59: Lineare Algebra
    Status VK 60: Analysis
    Status Wahrscheinlichkeitst

Gezeigt werden alle Foren bis zur Tiefe 2

Navigation
 Startseite...
 Neuerdings beta neu
 Forum...
 vorwissen...
 vorkurse...
 Werkzeuge...
 Nachhilfevermittlung beta...
 Online-Spiele beta
 Suchen
 Verein...
 Impressum
Das Projekt
Server und Internetanbindung werden durch Spenden finanziert.
Organisiert wird das Projekt von unserem Koordinatorenteam.
Hunderte Mitglieder helfen ehrenamtlich in unseren moderierten Foren.
Anbieter der Seite ist der gemeinnützige Verein "Vorhilfe.de e.V.".
Partnerseiten
Weitere Fächer:

Open Source FunktionenplotterFunkyPlot: Kostenloser und quelloffener Funktionenplotter für Linux und andere Betriebssysteme
Forum "Reelle Analysis mehrerer Veränderlichen" - Ableitung Polarkoordinaten
Ableitung Polarkoordinaten < mehrere Veränderl. < reell < Analysis < Hochschule < Mathe < Vorhilfe
Ansicht: [ geschachtelt ] | ^ Forum "Reelle Analysis mehrerer Veränderlichen"  | ^^ Alle Foren  | ^ Forenbaum  | Materialien

Ableitung Polarkoordinaten: Frage (beantwortet)
Status: (Frage) beantwortet Status 
Datum: 09:59 Mi 23.03.2011
Autor: Lentio

Aufgabe
aus x= [mm] pcos\alpha [/mm] und [mm] y=psin\alpha [/mm] erhält man [mm] \bruch{\partial f}{\partial p}=\bruch{\partial f}{\partial x}cos\alpha+\bruch{\partial f}{\partial y}sin\alpha [/mm]
[mm] \bruch{\partial f}{\partial \alpha}=-p \bruch{\partial f}{\partial x}sin\alpha+p \bruch{\partial f}{\partial y}cos\alpha [/mm]

Morgen!

Ich schaue mir gerade die Ableitungsregeln in Polarkoordinaten an und denke   :o ??? Verstehe z.B nicht, wie man mit dem Term [mm] \bruch{\partial f}{\partial x} [/mm] umgeht.

Und zwar wollte ich die Divergenz der Funktion= [mm] \pmat{ cos\alpha -sin\alpha\\ cos\alpha +sin\alpha } [/mm] berechnen. Das wäre doch [mm] div=\pmat{ \Delta p \\ \Delta \alpha}*\pmat{ cos\alpha -sin\alpha\\ cos\alpha +sin\alpha }. [/mm] Wie wende ich jetzt die Regel an?



mfg,

Lentio

        
Bezug
Ableitung Polarkoordinaten: Antwort
Status: (Antwort) fertig Status 
Datum: 10:30 Mi 23.03.2011
Autor: fred97


> aus x= [mm]pcos\alpha[/mm] und [mm]y=psin\alpha[/mm] erhält man
> [mm]\bruch{\partial f}{\partial p}=\bruch{\partial f}{\partial x}cos\alpha+\bruch{\partial f}{\partial y}sin\alpha[/mm]
>  
> [mm]\bruch{\partial f}{\partial \alpha}=-p \bruch{\partial f}{\partial x}sin\alpha+p \bruch{\partial f}{\partial y}cos\alpha[/mm]
>  
> Morgen!
>  
> Ich schaue mir gerade die Ableitungsregeln in
> Polarkoordinaten an und denke   :o ??? Verstehe z.B nicht,
> wie man mit dem Term [mm]\bruch{\partial f}{\partial x}[/mm]
> umgeht.


Hier kam die mehrdimensionale Kettenregel zum Zuge !

http://de.wikipedia.org/wiki/Verallgemeinerte_Kettenregel

>  
> Und zwar wollte ich die Divergenz der Funktion= [mm]\pmat{ cos\alpha -sin\alpha\\ cos\alpha +sin\alpha }[/mm]
> berechnen.

Die Divergenz ist nur für Funktionen [mm] \IR^n \to \IR^n [/mm] definiert !

Also: von welcher Funktion willst Du die Divergenz berechnen ?

FRED

> Das wäre doch [mm]div=\pmat{ \Delta p \\ \Delta \alpha}*\pmat{ cos\alpha -sin\alpha\\ cos\alpha +sin\alpha }.[/mm]
> Wie wende ich jetzt die Regel an?
>  
>
>
> mfg,
>  
> Lentio


Bezug
                
Bezug
Ableitung Polarkoordinaten: Frage (beantwortet)
Status: (Frage) beantwortet Status 
Datum: 10:54 Mi 23.03.2011
Autor: Lentio

Danke für die Antwort. Die Funktion geht von [mm] R^2-->R^2. [/mm]  Ich habe mir die wikiseite angeschaut , aber so ganz blicke ich immer noch nicht durch (eigentlich garnicht :( ).

mfg,

Lentio

Bezug
                        
Bezug
Ableitung Polarkoordinaten: Antwort
Status: (Antwort) fertig Status 
Datum: 11:01 Mi 23.03.2011
Autor: XPatrickX

Hallo,

du hast also die Funktion $f=f(x,y)$ gegeben. Dabei setzt du nun für [mm] x=p\cos\alpha [/mm] und für [mm] y=p\sin\alpha [/mm] ein. Damit erhälst du die verkettete Funktion [mm] f=f(p\cos\alpha,p\sin\alpha). [/mm]
Jetzt willst du nach p ableiten und somit kommt die Kettenregel ins Spiel. Zuerst musst du die äußere Funktion ableiten. Im ersten Argument der Funktion f steht ein p, sodass du zuerst [mm] \frac{\partial f}{\partial x}=f_x [/mm] bilden musst, die Nachableitung ist dann trivialerweise [mm] \cos\alpha. [/mm] Aber auch in der zweiten Komponente steht ein p, sodass du [mm] f_y [/mm] bilden musst mit anschließender Nachableitung.

Gruß Patrick

Bezug
                        
Bezug
Ableitung Polarkoordinaten: Antwort
Status: (Antwort) fertig Status 
Datum: 11:13 Mi 23.03.2011
Autor: fred97


> Danke für die Antwort. Die Funktion geht von [mm]R^2-->R^2.[/mm]  


Diese aber nicht: [mm] \alpha \to [/mm]  $ [mm] \pmat{ cos\alpha -sin\alpha\\ cos\alpha +sin\alpha } [/mm] $

FRED


> Ich habe mir die wikiseite angeschaut , aber so ganz blicke
> ich immer noch nicht durch (eigentlich garnicht :( ).
>  
> mfg,
>  
> Lentio


Bezug
                                
Bezug
Ableitung Polarkoordinaten: Frage (beantwortet)
Status: (Frage) beantwortet Status 
Datum: 11:39 Mi 23.03.2011
Autor: Lentio

Vielleicht sollte ich doch lieber die gesammte Aufgabe darstellen. Sieht so aus, als ob es nicht nur an der Kettenregel hapert.
Es ist gegeben eine Funktion f: [mm] R^2-->R^2, \vektor{x \\ y} -->\vektor{x - y\\ x+y}. [/mm] F soll nun in Polarkoordinaten gegeben werden und in diesen Koordinaten soll div(f) bestimmt werden.
Mit [mm] x=pcos\alpha [/mm] und y= [mm] psin\alpha [/mm]
f(p, [mm] \alpha)=\pmat{ pcos\alpha - psin\alpha\\ pcos\alpha + psin\alpha } =p*\pmat{ cos\alpha - sin\alpha\\ cos\alpha + sin\alpha }. [/mm] Stimmt das denn soweit?
Die Divergenz würde sich jetzt doch berechnen mit [mm] \bruch{\partial f}{\partial p}(pcos\alpha-psin\alpha)+\bruch{\partial f}{\partial \alpha}(pcos\alpha+psin\alpha). [/mm] Also das mit der Kettenregel will mir einfach nicht in den Kopf.

mfg,

Lentio

Bezug
                                        
Bezug
Ableitung Polarkoordinaten: Antwort
Status: (Antwort) fertig Status 
Datum: 11:52 Mi 23.03.2011
Autor: MathePower

Hallo Lentio,

> Vielleicht sollte ich doch lieber die gesammte Aufgabe
> darstellen. Sieht so aus, als ob es nicht nur an der
> Kettenregel hapert.
>  Es ist gegeben eine Funktion f: [mm]R^2-->R^2, \vektor{x \\ y} -->\vektor{x - y\\ x+y}.[/mm]
> F soll nun in Polarkoordinaten gegeben werden und in diesen
> Koordinaten soll div(f) bestimmt werden.
>  Mit [mm]x=pcos\alpha[/mm] und y= [mm]psin\alpha[/mm]
>   f(p, [mm]\alpha)=\pmat{ pcos\alpha - psin\alpha\\ pcos\alpha + psin\alpha } =p*\pmat{ cos\alpha - sin\alpha\\ cos\alpha + sin\alpha }.[/mm]
> Stimmt das denn soweit?


Ja.


>  Die Divergenz würde sich jetzt doch berechnen mit
> [mm]\bruch{\partial f}{\partial p}(pcos\alpha-psin\alpha)+\bruch{\partial f}{\partial \alpha}(pcos\alpha+psin\alpha).[/mm]


Auch das ist richtig.


> Also das mit der Kettenregel will mir einfach nicht in den
> Kopf.
>  
> mfg,
>  
> Lentio


Gruss
MathePower

Bezug
                                                
Bezug
Ableitung Polarkoordinaten: Frage (beantwortet)
Status: (Frage) beantwortet Status 
Datum: 12:08 Mi 23.03.2011
Autor: Lentio


> >  Die Divergenz würde sich jetzt doch berechnen mit

> > [mm]\bruch{\partial f}{\partial p}(pcos\alpha-psin\alpha)+\bruch{\partial f}{\partial \alpha}(pcos\alpha+psin\alpha).[/mm]
> >



Aber offensichtlich ist das nicht einfach: = [mm] cos\alpha-sin\alpha-psin\alpha+pcos\alpha [/mm]

>
> > Also das mit der Kettenregel will mir einfach nicht in den
> > Kopf.


Tut mir leid, ich kann mich aber nur wiederholen.


mfg,

Lentio

Bezug
                                                        
Bezug
Ableitung Polarkoordinaten: Frage (beantwortet)
Status: (Frage) beantwortet Status 
Datum: 14:47 Mi 23.03.2011
Autor: Lentio

Betrachtet man die Divergenz  in kartesischen Koordinaten ergibt das 2. Dementsprechend muss das Ergebnis in Polark. sein. Wie schafft mann es denn, die ganzen cos/sinus "loszuwerden"?


mfg,

Lentio

Bezug
                                                                
Bezug
Ableitung Polarkoordinaten: Antwort
Status: (Antwort) fertig Status 
Datum: 23:07 Do 24.03.2011
Autor: Denny22

1. kartesische Koordinaten: Sei [mm] $f:\IR^2\rightarrow\IR^2$ [/mm] mit [mm] $f\vektor{x_1 \\ x_2}=\vektor{x_1-x_2 \\ x_1+x_2}=:\vektor{f_1(x_1,x_2) \\ f_2(x_1,x_2)}$. [/mm] Allgemein ist die Divergenz (in zweidimensionalen kartesischen Koordinaten) gegeben durch

[mm] $\left(\mathrm{div}\,f\right)\vektor{x_1 \\ x_2}=\left(\nabla\bullet f\right)\vektor{x_1 \\ x_2}=\left(\frac{\partial}{\partial x_1},\frac{\partial}{\partial x_2}\right)\cdot\vektor{f_1(x_1,x_2) \\ f_2(x_1,x_2)}=\frac{\partial}{\partial x_1}f_1(x_1,x_2)+\frac{\partial}{\partial x_2}f_2(x_1,x_2)=\sum_{i=1}^2\frac{\partial}{\partial x_i}f_i(x_1,x_2)$ [/mm]

Wegen

[mm] $\frac{\partial}{\partial x_1}f_1(x_1,x_2)=1$ [/mm]
[mm] $\frac{\partial}{\partial x_2}f_2(x_1,x_2)=1$ [/mm]

ergibt sich

[mm] $\left(\mathrm{div}\,f\right)\vektor{x_1 \\ x_2}=1+1=2$ [/mm]

2. Polarkoordinaten: Sei [mm] $f:\IC\rightarrow\IC$ [/mm] mit [mm] $f(r,\varphi)=r\left(\left(\cos\varphi-\sin\varphi\right)+i\left(\cos\varphi+\sin\varphi\right)\right)$. [/mm] Definiere [mm] $f_1(r,\varphi):=\mathrm{Re}\,f(r,\varphi)$ [/mm] und [mm] $f_2(r,\varphi):=\mathrm{Im}\,f(r,\varphi)$. [/mm] Allgemein ist die Divergenz (in Polarkoordinaten) gegeben durch

[mm] $\left(\mathrm{div}\,f\right)(r,\varphi)=\left(\nabla\bullet f\right)(r,\varphi)=\left[\frac{\partial}{\partial r}f_1(r,\varphi)\right]\cos\varphi-r\left[\frac{\partial}{\partial \varphi}f_1(r,\varphi)\right]\sin\varphi+\left[\frac{\partial}{\partial r}f_2(r,\varphi)\right]\sin\varphi+r\left[\frac{\partial}{\partial \varphi}f_2(r,\varphi)\right]\cos\varphi$ [/mm]

Wegen

[mm] $\frac{\partial}{\partial r}f_1(r,\varphi)=\cos\varphi-\sin\varphi$ [/mm]
[mm] $\frac{\partial}{\partial \varphi}f_1(r,\varphi)=-\sin\varphi-\cos\varphi$ [/mm]
[mm] $\frac{\partial}{\partial r}f_2(r,\varphi)=\cos\varphi+\sin\varphi$ [/mm]
[mm] $\frac{\partial}{\partial \varphi}f_2(r,\varphi)=-\sin\varphi+\cos\varphi$ [/mm]

ergibt sich aus der Eulerschen Formel

[mm] $\left(\mathrm{div}\,f\right)(r,\varphi)=\cos^2\varphi+\sin^2\varphi+r\left(\cos^2\varphi+\sin^2\varphi\right)=1+r$ [/mm]

Leider ist es etwas spät und ich kann mich gerade nicht vernümpftig konzentrieren. Aus irgendeinem Grund müssen diese Ergebnisse übereinstimmen. Ich weiß nur gerade nicht warum. Vielleicht fällt es Dir oder jemanden anderen ein.

Viel Glück noch.

Bezug
                                                                        
Bezug
Ableitung Polarkoordinaten: Mitteilung
Status: (Mitteilung) Reaktion unnötig Status 
Datum: 00:50 Fr 25.03.2011
Autor: Al-Chwarizmi


> 1. kartesische Koordinaten: Sei [mm]f:\IR^2\rightarrow\IR^2[/mm] mit
> [mm]f\vektor{x_1 \\ x_2}=\vektor{x_1-x_2 \\ x_1+x_2}=:\vektor{f_1(x_1,x_2) \\ f_2(x_1,x_2)}[/mm].
> Allgemein ist die Divergenz (in zweidimensionalen
> kartesischen Koordinaten) gegeben durch
>  
> [mm]\left(\mathrm{div}\,f\right)\vektor{x_1 \\ x_2}=\left(\nabla\bullet f\right)\vektor{x_1 \\ x_2}=\left(\frac{\partial}{\partial x_1},\frac{\partial}{\partial x_2}\right)\cdot\vektor{f_1(x_1,x_2) \\ f_2(x_1,x_2)}=\frac{\partial}{\partial x_1}f_1(x_1,x_2)+\frac{\partial}{\partial x_2}f_2(x_1,x_2)=\sum_{i=1}^2\frac{\partial}{\partial x_i}f_i(x_1,x_2)[/mm]
>  
> Wegen
>  
> [mm]\frac{\partial}{\partial x_1}f_1(x_1,x_2)=1[/mm]
>  
> [mm]\frac{\partial}{\partial x_2}f_2(x_1,x_2)=1[/mm]
>  
> ergibt sich
>
> [mm]\left(\mathrm{div}\,f\right)\vektor{x_1 \\ x_2}=1+1=2[/mm]
>  
> 2. Polarkoordinaten: Sei [mm]f:\IC\rightarrow\IC[/mm] mit
> [mm]f(r,\varphi)=r\left(\left(\cos\varphi-\sin\varphi\right)+i\left(\cos\varphi+\sin\varphi\right)\right)[/mm].
> Definiere [mm]f_1(r,\varphi):=\mathrm{Re}\,f(r,\varphi)[/mm] und
> [mm]f_2(r,\varphi):=\mathrm{Im}\,f(r,\varphi)[/mm]. Allgemein ist
> die Divergenz (in zweidimensionalen Polarkoordinaten)
> gegeben durch
>  
> [mm]\left(\mathrm{div}\,f\right)(r,\varphi)=\left(\nabla\bullet f\right)(r,\varphi)=\left[\frac{\partial}{\partial r}f_1(r,\varphi)\right]\cos\varphi-r\left[\frac{\partial}{\partial \varphi}f_1(r,\varphi)\right]\sin\varphi+\left[\frac{\partial}{\partial r}f_2(r,\varphi)\right]\sin\varphi+r\left[\frac{\partial}{\partial \varphi}f_2(r,\varphi)\right]\cos\varphi[/mm]
>  
> Wegen
>  
> [mm]\frac{\partial}{\partial r}f_1(r,\varphi)=\cos\varphi-\sin\varphi[/mm]
>  
> [mm]\frac{\partial}{\partial \varphi}f_1(r,\varphi)=-\sin\varphi-\cos\varphi[/mm]
>  
> [mm]\frac{\partial}{\partial r}f_2(r,\varphi)=\cos\varphi+\sin\varphi[/mm]
>  
> [mm]\frac{\partial}{\partial \varphi}f_2(r,\varphi)=-\sin\varphi+\cos\varphi[/mm]
>  
> ergibt sich aus der Eulerschen Formel
>  
> [mm]\left(\mathrm{div}\,f\right)(r,\varphi)=\cos^2\varphi+\sin^2\varphi+r\left(\cos^2\varphi+\sin^2\varphi\right)=1+r[/mm]
>  
> Leider ist es etwas spät und ich kann mich gerade nicht
> vernümpftig konzentrieren. Aus irgendeinem Grund müssen
> diese Ergebnisse übereinstimmen. Ich weiß nur gerade
> nicht warum. Vielleicht fällt es Dir oder jemanden anderen
> ein.
>
> Viel Glück noch.


Hallo Denny22,

mit deiner Formel für die Berechnung der Divergenz in
Polarkoordinaten verfolgst du genau die Idee, die ich
schon angedeutet hatte, alles mittels Polarkoordinaten
darzustellen.
Das wird aber, wie ich auch schon festgestellt habe, ziemlich
umständlich. Zum Beispiel muss man dabei aufpassen, dass
die "neuen" Koordinaten r und [mm] \varphi [/mm] nicht mit jenen überein-
stimmen, die man zuerst als Polarkoordinaten anstelle der
rechtwinkligen Koordinaten (x,y) eingeführt hat. Stellt man
den Vektor [mm] \pmat{x-y\\x+y} [/mm] in Polarkoordinaten dar, so hat man

    $\ [mm] \pmat{x-y\\x+y}\ [/mm] =\ [mm] R*\pmat{cos(\Phi)\\sin(\Phi)}$ [/mm]

Dabei ist z.B.  [mm] R=\sqrt{2}*r [/mm]  und [mm] tan(\Phi)=\frac{cos(\varphi)+sin(\varphi)}{cos(\varphi)-sin(\varphi)} [/mm]

Damit wird doch alles recht kompliziert, und ich frage mich,
ob dieser mühsame "Umweg" zur einfachen, schon von vorn-
herein bekannten Lösung wirklich der Sinn der Aufgabe sein
sollte.

LG    Al-Chw.

Bezug
                                                        
Bezug
Ableitung Polarkoordinaten: Antwort
Status: (Antwort) fertig Status 
Datum: 18:19 Mi 23.03.2011
Autor: MathePower

Hallo Lentio,

> > >  Die Divergenz würde sich jetzt doch berechnen mit

> > > [mm]\bruch{\partial f}{\partial p}(pcos\alpha-psin\alpha)+\bruch{\partial f}{\partial \alpha}(pcos\alpha+psin\alpha).[/mm]
> > >
>  
>
>
> Aber offensichtlich ist das nicht einfach: =
> [mm]cos\alpha-sin\alpha-psin\alpha+pcos\alpha[/mm]
>  >

> > > Also das mit der Kettenregel will mir einfach nicht in den
> > > Kopf.


Aus den Gleichungen

[mm]\bruch{\partial f}{\partial p}=\bruch{\partial f}{\partial x}cos\alpha+\bruch{\partial f}{\partial y}sin\alpha [/mm]
[mm]\bruch{\partial f}{\partial \alpha}=-p \bruch{\partial f}{\partial x}sin\alpha+p \bruch{\partial f}{\partial y}cos\alpha[/mm]

ist zunächst [mm]\bruch{\partial f}{\partial x}, \ \bruch{\partial f}{\partial y}[/mm] zu bestimmen.

Danach kannst Du die DIvergent ausrechnen:

[mm]\operatorname{div} \ f=\bruch{\partial f}{\partial x}+\bruch{\partial f}{\partial y}[/mm]


>  
>
> Tut mir leid, ich kann mich aber nur wiederholen.
>  
>
> mfg,
>  
> Lentio


Gruss
MathePower

Bezug
                                        
Bezug
Ableitung Polarkoordinaten: Antwort
Status: (Antwort) fertig Status 
Datum: 15:42 Mi 23.03.2011
Autor: Al-Chwarizmi


> Vielleicht sollte ich doch lieber die gesamte Aufgabe
> darstellen. Sieht so aus, als ob es nicht nur an der
> Kettenregel hapert.
>  Es ist gegeben eine Funktion f: [mm]R^2-->R^2, \vektor{x \\ y} -->\vektor{x - y\\ x+y}.[/mm]
> F soll nun in Polarkoordinaten gegeben werden und in diesen
> Koordinaten soll div(f) bestimmt werden.

Mit $ [mm] x=p*cos(\alpha) [/mm] $ und y= $ [mm] p*sin(\alpha) [/mm] $

f(p, $ [mm] \alpha)=\pmat{ p*cos(\alpha) - p*sin(\alpha)\\ p*cos(\alpha) + p*sin(\alpha) } =p\cdot{}\pmat{ cos(\alpha) - sin(\alpha)\\ cos(\alpha) + sin(\alpha) }. [/mm] $


Soll die Funktion komplett in Polarkoordinaten
dargestellt werden ?

Was du notiert hast, ist eine Darstellung halb in
Polarkoordinaten, halb in rechtwinkligen.

LG    Al-Chw.

Bezug
                                                
Bezug
Ableitung Polarkoordinaten: Frage (überfällig)
Status: (Frage) überfällig Status 
Datum: 15:57 Mi 23.03.2011
Autor: Lentio

Also in der Aufgabenstellung wird gesagt, die "Funktion soll in Polarkoordinaten gegeben werden ". Jetzt haben wir bei einer ähnlichen Aufgabe auch nur x und y mit den entsprechenden Terme ersetzt.

mfg,

Lentio

Bezug
                                                        
Bezug
Ableitung Polarkoordinaten: Fälligkeit abgelaufen
Status: (Mitteilung) Reaktion unnötig Status 
Datum: 16:20 Fr 25.03.2011
Autor: matux

$MATUXTEXT(ueberfaellige_frage)
Bezug
Ansicht: [ geschachtelt ] | ^ Forum "Reelle Analysis mehrerer Veränderlichen"  | ^^ Alle Foren  | ^ Forenbaum  | Materialien


^ Seitenanfang ^
www.vorkurse.de
[ Startseite | Mitglieder | Teams | Forum | Wissen | Kurse | Impressum ]